For what value of K ,the matrix [2-K 3 ] is not invertible?

Your question is not clear and appears to be unclear but still I am providing you link to a similar query:

https://www.meritnation.com/ask-answer/question/for-what-value-of-k-the-matrix-2-k-3-is-not-invertib/matrices/5512079

  • -1
What are you looking for?